chiarimento moltiplicatori lagrange

Messaggioda andros » 25/01/2015, 09:08

Buongiorno ragazzi.....ho un dubbio:
se ho una funzione$f(x,y,z)$ e il mio vincolo è del tipo $E={(x,y,z)in RR^3:x^2+y^2<=1, |z|<=1}$
dovrei impostare la funzione lagrangiana del tipo $ L(x,y,z,\mu,\lambda)= f(x,y,z)- \mu(x^2+y^2-1)-\lambda(??)$ ma come devo comportarmi col valore assoluto ?

E se invece trovo $z<0$ devo comunque metterlo nella lagrangiana (cioà con $\lambda(z-1)$) o mi basta più semplicemente considerare solo i punti con $z<0$ ?
Ultima modifica di andros il 25/01/2015, 17:19, modificato 1 volta in totale.
andros
Junior Member
Junior Member
 
Messaggio: 131 di 288
Iscritto il: 05/10/2013, 16:09

Re: chiarimento moltiplicatori lagranga

Messaggioda dissonance » 25/01/2015, 11:17

Ti conviene farti furbo e riscrivere \(\lvert z \rvert \le 1\) come \(z^2 \le 1\).
dissonance
Moderatore
Moderatore
 
Messaggio: 11489 di 27760
Iscritto il: 24/05/2008, 19:39
Località: Nomade

Re: chiarimento moltiplicatori lagranga

Messaggioda andros » 25/01/2015, 15:09

quindi dovrei considerare due sistemi in realtà : il primo con $z=1$ e il secondo con $z=-1$?
andros
Junior Member
Junior Member
 
Messaggio: 132 di 288
Iscritto il: 05/10/2013, 16:09

Re: chiarimento moltiplicatori lagrange

Messaggioda Noisemaker » 26/01/2015, 09:15

la funzione qual è?
Dobbiamo Sapere, e Sapremo
Avatar utente
Noisemaker
Cannot live without
Cannot live without
 
Messaggio: 1967 di 4050
Iscritto il: 11/12/2011, 19:23

Re: chiarimento moltiplicatori lagrange

Messaggioda andros » 26/01/2015, 10:10

Per esempio con queste funzioni e rispettivi vincoli che lagrangiana e che quindi che sistemi dovrei impostare ? tu come faresti ?
Immagine
Grazie 1000 per l'aiuto :D
andros
Junior Member
Junior Member
 
Messaggio: 133 di 288
Iscritto il: 05/10/2013, 16:09

Re: chiarimento moltiplicatori lagrange

Messaggioda Noisemaker » 26/01/2015, 19:56

Come esempio,determininiamo, se esistono,
\[\max_{C} f,\qquad\min_{C}f,\]
dove \[f(x,y,z):= x^2-2y^2+xz,\qquad\mbox{e} \qquad C:=\{{(x,y,z)}\in\mathbb{R}^3: x^2+y^2\le1,\,\,|z|\le1\}.\]

Si può osservare inizialmente che la funzione $f$ è di classe $C^{\infty}(\RR^3)$ e l'insieme $C,$ che rappresenta la regione dello spazio generata dall'intersezione tra il cilindro infinito con asse di simmetria coincidente con l'asse $z$ e i piani paralleli al piano $xy$ di equazione $z=1$ e $z=-1,$ è un insieme compatto cioè chiuso e limitato, avendosi evidentemente le limitazioni
\[|x|\le 1,\quad|y|\le 1,\quad|z|\le 1,\]
pertanto il teorema di Weierstrass assicura l'esistenza del massimo e del minimo assoluto della fuzione $f$ sull'insieme $C.$ Tali punti vanno cercati tra i punti inteni all'insieme $C$ dove il gradiente di $f$ si annulla, e sulla frontiera di $C;$ per i punti interni si ha
\[\nabla f=(2x+z;-4y;x)=(0,0,0)\qquad\Leftrightarrow\quad (x,y,z,)=(0,0,0),\]
ossia l'unico punto interno a $C$ è l'origine. Per stabilirne la natura, si può verificare facilmente che non si tratta né di un massimo né di un minimo per $f,$ in quanto restringendo ad esempio la funzione alla funzione
\[f(x,y,x)=x^2-2y^2+x^2=2x^2-2y^2=2(x^2-y^2),\]
è evidente che l'origine è un punto sella. Allora, i massmi e minimi garantiti dal teorema di Weierstrass, si troverranno necessariamente sulla frontiera di $C.$ La frontiera di $C$ è composta dalla superficie laterale del cilindro e dalle due basi, quindi si tratterà di studiare i seguenti problemi di massimo e minimo:
  • $ f_1(x,y,1)=x^2+x-2y^2,$ relativamente all'insieme $$C_1:=\{ (x,y,z)\in\mathbb{R}^3: x^2+y^2\le1,\,\,z=1\},$$
  • $ f_2(x,y,-1)=x^2-x-2y^2,$ relativamente all'insieme $$C_2:=\{ (x,y,z)\in\mathbb{R}^3: x^2+y^2\le1,\,\,z=-1\},$$
  • $ f_3(x,y,z)=x^2 -2y^2+xz,$ relativamente all'insieme $$C_3:=\{ (x,y,z)\in\mathbb{R}^3: x^2+y^2=1,\,\,|z|<1\},.$$
Per la funzione $f_1,$ cosi come per la funzione $f_2,$ abbiano a che fare con lo stusio di una funzione in due variabili; essendo $f_1$ ($f_2$) ci classe $C^{\infty}(\RR^2)$ e l'insieme $C_1$ compatto, come prima per il teorema di Weierstrass, massimi e minimi di $f_1$ ($f_2$) su $C_1$ ($C_2$) esistono ed adranno cercati tra i punti interni a $C_1$ ($C_2$) dove i gradiente di $f_1$ ($f_2$) si annulla o sulla frontiera di $C_1$ ($C_2$). Per i punti interni, rispettivamente avremo:
\begin{align}
\nabla f_1&=(2x+1;-4y)=(0,0)\quad\Leftrightarrow\quad (x,y)=\left(-\frac{1}{2},0\right)\\
\nabla f_2&=(2x-1;-4y)=(0,0)\quad\Leftrightarrow\quad (x,y)=\left( \frac{1}{2},0\right);
\end{align}
anche in questo caso, si può verificare facilmente che i punti trovato no sono, per le rispettive funzioni, né di un massimo né di un minimo (restringendo ad esempio le funzioni alle funzioni $f_1(x,0)$ e $f_1(-1/2,y)$ e $f_2(x,0)$ e $f_2( 1/2,y)$), quindi i punti di max e min garantiti dal teorema di Weierstrass si troveranno sulla frontiera di $C_1$ ($C_2$); su può applicare il teorema dei moltiplicatori di Lagrange, essendone verficate le ipotesi: infatti detta $g (x,y):= x^2+y^2-1,$ evidentemente $g$ è di classe $C^{\infty}(\RR^2),$ essendo $\nabla g=(2x,2y)=0\Leftrightarrow(x,y)=(0,0)$ e$g(0,0)\ne0,$ le ipotesi sono verificate. Allora deve esserre rispettivamente:
\begin{align}
\nabla f_1-\lambda \nabla g=(0,0)&\quad\Leftrightarrow\quad (2x+1;-4y)-\lambda(2x,2y)=(0,0)\\
\nabla f_2-\lambda \nabla g=(0,0)&\quad\Leftrightarrow\quad (2x-1;-4y)-\lambda(2x,2y)=(0,0),
\end{align}
che equivale ai due sistemi:
\begin{align}
\Sigma_1:&\begin{cases}
2x+1-2\lambda x=0\\
-4y-2\lambda y=0\\
x^2+y^2-1=0
\end{cases}=\begin{cases}
2x+1-2\lambda x=0\\
y =0\\
x^2 =1
\end{cases} \cup\begin{cases}
2x+1-2\lambda x=0\\
\lambda=-2\\
x^2+y^2-1=0
\end{cases} \\
&A\left(1,0\right),\quad B\left(-1,0\right),\quad C\left(-\frac{1}{6},-\frac{\sqrt{35}}{6}\right),\quad D\left(-\frac{1}{6}, \frac{\sqrt{35}}{6}\right)\\
\Sigma_2:&\begin{cases}
2x-1-2\lambda x=0\\
-4y-2\lambda y=0\\
x^2+y^2-1=0
\end{cases}\begin{cases}
2x+1-2\lambda x=0\\
y =0\\
x^2 =1
\end{cases} \cup\begin{cases}
2x-1-2\lambda x=0\\
\lambda=-2\\
x^2+y^2-1=0
\end{cases} \\
&E\left(1,0\right),\quad F\left(-1,0\right),\quad G\left( \frac{1}{6},-\frac{\sqrt{35}}{6}\right),\quad H\left( \frac{1}{6}, \frac{\sqrt{35}}{6}\right);
\end{align}
allora la funzione di tre variabili sulle basi del cilindro avrà i candidati punti di massimo e minimo:
\begin{align}
&A\left(1,0,1\right), \quad B\left(-1,0,1\right),\quad C\left(-\frac{1}{6},-\frac{\sqrt{35}}{6},1\right),\quad D\left(-\frac{1}{6}, \frac{\sqrt{35}}{6},1\right),\\
& E\left(1,0,-1\right),\quad F\left(-1,0,-1\right),\quad G\left( \frac{1}{6},-\frac{\sqrt{35}}{6},-1\right),\quad H\left( \frac{1}{6}, \frac{\sqrt{35}}{6},-1\right),
\end{align}

Rimane da considerare la funzione $f_3$ sull'insieme $C_3:$ possiamo applicare il teorema dei moltiplicatori, poichè anche in questo caso le ipotesi sono verificate (controlla!); allora dovrà essere:
\begin{align}
\nabla f_3-\lambda \nabla g=(0,0,0)&\quad\Leftrightarrow\quad (2x+z;-4y;x)-\lambda(2x,2y,0)=(0,0,0)
\end{align}
che equivale al sistema:
\begin{align}
\Sigma_1:&\begin{cases}
2x+z-2\lambda x=0\\
-4y-2\lambda y=0\\
x=0\\
x^2+y^2-1=0\\
|z|<1
\end{cases} =\begin{cases}
z =0\\
-4y-2\lambda y=0\\
x=0\\
y^2-1=0\\
|z|<1
\end{cases} =\begin{cases}
x =0\\
y=\pm1\\
z=0
\end{cases} ,
\end{align}
quindi si trovano gli ulteriori punti:
\begin{align}
I\left(0,1,0\right),&\quad L\left(0,-1,0\right).
\end{align}
Calcolando ora il valore della funzione nei punti trovati si conclude che
\begin{align}
&f\left(A\right)= f\left(B\right)=2,\quad f\left(C\right)= f\left(G\right)=-\frac{75}{36},\quad f\left(D\right)= f\left(H\right)=\frac{75}{36}, f\left(E\right)=f\left(F\right)=0,\quad f\left( I\right)=f\left( L\right)=-2,
\end{align}
pertanto si conclude
\[\max_{C} f=\frac{75}{36},\qquad\min_{C}f=-\frac{75}{36}.\]
Dobbiamo Sapere, e Sapremo
Avatar utente
Noisemaker
Cannot live without
Cannot live without
 
Messaggio: 1968 di 4050
Iscritto il: 11/12/2011, 19:23


Torna a Analisi matematica di base

Chi c’è in linea

Visitano il forum: Nessuno e 1 ospite